The set of all numbers greater than or equal to −5 and less than 5.

Answers

Answer 1

Answer:

No. greater than of equal to (-5) = -4,-3,-2,-1,0,1,2,3,4,5,6...........

No. less than 5 = 4,3,2,1 ,0,-1......

{ -4,-3,-2,-1,0,1,2,3,4}


Related Questions

Follow the steps to find the area of the shaded region.


First use the formula below to find the area of the whole sector.


Sector Area= ( angle of sector/360). R2


Sector Area = ? Cm^2
Round to four decimal places.

14 cm

46°

14 cm

Answers

The area of the sector, to 4 decimal places, is 78.6794 cm².

We have given that,

Sector Area= ( angle of sector/360). R2

We have to determine the Sector Area

What is the Area of a Sector of a Circle?

Area of sector = ∅/360 × πr².

We have given the following:

∅ = 46°

Radius (r) = 14 cm

Area of sector = 46/360 × π(14²)

Area of sector ≈ 78.6794 cm²

The area of the sector is approximately 78.6794 cm².

Learn more about the area of the sector on:

brainly.com/question/8159268

#SPJ1

Analyze the diagram below and complete the instructions. Find the value of x and the value of y

Answers

The value of the variable x and y will be 15 and 5√3. Then the correct option is D.

The complete question is attached below.

What is trigonometry?

The connection between the lengths and angles of a triangular shape is the subject of trigonometry.

The right triangle is shown.

Then the value of x will be

x = 10√3 × sin 60°

x = 15

Then the value of y will be

y = 10√3 × cos 60°

y = 5√3

Then the correct option is D.

More about the trigonometry link is given below.

https://brainly.com/question/22698523

#SPJ1

Which of the following terms best describes a vertical line that the graph of a function approaches but never intersects

Answers

Answer: an asymptote is, essentially, a line that a graph approaches , but     does not intersect.      

Step-by-step explanation:

for example , in the following graph of y =1x y=1x the line approaches the x- axis (y=0) but never touches it

Describe the change in the graph of the parabola f(x) when it transforms into g(x) =
The parabola g(x) will open in the opposite direction of f(x), and the parabola will be narrower than f(x).
The parabola g(x) will open in the same direction of f(x), and the parabola will be narrower than f(x).
The parabola g(x) will open in the opposite direction of f(x), and the parabola will be wider than f(x).
The parabola g(x) will open in the same direction of f(x), and the parabola will be wider than f(x).

Answers

Answer:

  (d)  The parabola g(x) will open in the same direction of f(x), and the parabola will be wider than f(x).

Step-by-step explanation:

We assume you intend ...

  f(x) = equation of a parabola

  g(x) = 2/3·f(x)

Multiplying a function by a factor of 2/3 will cause it to be compressed vertically to 2/3 of its original height. When the function is a parabola, this has the effect of making it appear wider than before the compression.

__

The compression factor is positive, so points on the graph remain on the same side of the x-axis. The direction in which the graph opens is not changed.

The attachment shows parabolas that open upward and downward, along with the transformed version.

Answer:

D.)  The parabola g(x) will open in the same direction of f(x), and the parabola will be wider than f(x).

Step-by-step explanation:

I got it right on the test :)

stay hydrated.

Word Problem 3 Priscilla works 43 hours a week for eight weeks. If she makes $14.73 per hour, how much money did she make in the eight weeks? Operation(s): Math sentence(s): Solution: Word Problem 3 Priscilla works 43 hours a week for eight weeks . If she makes $ 14.73 per hour , how much money did she make in the eight weeks ? Operation ( s ) : Math sentence ( s ) : Solution :​

Answers

Answer:

$5067,12

Step-by-step explanation:

14,73*43*8

The required money Priscilla will earn in 8 weeks is $5067.12.

As mentioned in the question, Priscilla works 43 hours a week for eight weeks. If she makes $14.73 per hour,

What is simplification?

The process in mathematics to operate and interpret the function to make the function or expression simple or more understandable is called simplifying and the process is called simplification.

Here,
Let the amount earned be x,
According to the condition,
x = 8 × 43 × 14.73
x =  $5067.12.

Thus, the required money Priscilla will earn in 8 weeks is $5067.12.

Learn more about simplification here:

https://brainly.com/question/12501526

#SPJ2

Tan + Cos / 1 + Sin = Sec

Answers

[tex]\tan x+\frac{\cos x}{1+\sin x}\\\\ =\frac{\sin x}{\cos x}+\frac{\cos x}{1+\sin x}\\\\=\frac{\sin x(1+\sin x)+\cos^{2} x}{\cos x(1+\sin x)}\\\\=\frac{\sin^{2} x+\cos^{2} x+\sin x}{\cos x(1+\sin x)}\\\\=\frac{1+\sin x}{\cos x(1+\sin x)}\\\\=\frac{1}{\cos x}\\\\=\sec x[/tex]

Select The correct answer.
Segment AB Is tangent to the circle at point B. Which equation describes the relationship between the tangent and secant line segments?
B
D
OA
(AD)² = (AC) (AB)
OB. (AB)2 = (AC)(CD)
OC. (AB)2 = (AC)(AD)
OD. AB=(AC + AD)

Answers

According to the tangent-secant theorem, the relationship of the tangent and secant in the given circle is: C. AB² = (AC)(AD).

What is the Tangent-Secant Theorem?

The secant tangent theorem defines the relationship between the lengths of the secant and the tangent line segments when they meet outside a circle.

Based on the tangent-secant theorem, using the image given, the equation that describes the relationship of the tangent and secant in the given circle is:

C. AB² = (AC)(AD)

Learn more about the tangent-secant theorem on:

https://brainly.com/question/4494553

#SPJ1

Answer: C

Step-by-step explanation:

got it right on my test!!

urgent help wanted/ needed

Answers

The equation in slope-intercept form is: y = -x + 5

The equation in point-slope form is: y - 3 = -1(x - 2)

How to Find the Equation of a Line in Point-Slope Form and Slope-intercept Form?Where (a, b) is a point on the line and m is the slope of the line, the equation in point-slope form for such line would be: y - b = m(x - a).Where b = y-intercept and m = slope, the slope-intercept form for the equation of the line is expressed as: y = mx + b.

Slope (m) = rise/run = -(2/2) = -1

Y-intercept (b) = 5 (where the line intercepts the y-axis)

Using a point, (2, 3) = (a, b) and m = -1, the equation in point-slope form is:

y - 3 = -1(x - 2)

Rewrite in slope-intercept form:

y - 3 = -x + 2

y = -x + 2 + 3

y = -x + 5

Learn more about point-slope and slope-intercept form on:

https://brainly.com/question/24907633

#SPJ1

The equation for the pH of a substance is pH = -log[H*], where H+ is the concentration of hydrogen ions. A basic
solution has a pH of 11.2. An acidic solution has a pH of 2.4. What is the approximate difference in the concentratic
of hydrogen ions between the two solutions?
1.6x10-9
O4.0x10-3
O 6.7x10-1
O 1.6x10¹11

Answers

The approximate difference in the concentration of hydrogen ions between the two solutions is 4.0×10⁻³

What is pH value?

The pH of a solution is a measurement of the concentration of hydrogen ions in the solution, as well as its acidity or alkalinity. Normally, the pH scale ranges from 0 to 14.

pH value for basic solution = 11.2

pH value for acidic solution = 2.4

Concentration for a basic solution:

[tex]\rm = 10^{-11.2}[/tex]

[tex]= 6.309\times10^{-12}[/tex]

Concentration for an acidic solution:

[tex]\rm = 10^{-2.4}[/tex]

[tex]= 3.981\times10^{-3}[/tex]

Difference between the concentration:

[tex]= 3.981\times10^{-3}- 6.309\times10^{-12}[/tex]

[tex]= 3.98\times10^{-3}[/tex]

[tex]= 4\times10^{-3}[/tex]

Thus, the approximate difference in the concentration of hydrogen ions between the two solutions is 4.0×10⁻³

Learn more about the pH value here:

brainly.com/question/1503669

#SPJ1

what is (x^2y^3)^1/3 / 3 sqrt x^2

Answers

Answer:  y/(3x^1/3)

Step-by-step explanation:

A 13 ​-ft ladder is leaning against a house when its base starts to slide away. By the time the base is 12 ft from the​ house, the base is moving away at the rate of 15 ​ft/sec.
a. What is the rate of change of the height of the top of the​ ladder?
b. At what rate is the area of the triangle formed by the​ ladder, wall, and ground changing​ then?
c. At what rate is the angle between the ladder and the ground changing​ then?

Answers

I think the answer is B because all the air other answers are incorrect

Christmas bulbs made of different colours are set to light after 8seconds,10 seconds and 14 seconds. How many times will they light simultaneously in one hour if they start together​

Answers

Answer:

36 seconds will be the answer... by looking down here↓:

Step-by-step explanation:

6 secs x 6 cycles = 36 seconds

9 secs x 4 cycles = 36 secs

12 ses x 3 cycles = 36 secs

how do you simplify

−(12st3+34s3t+t2−1)

Answers

Answer:

-34s^3t - 12st^3 - t^2 + 1

Step-by-step explanation:

hope this helps

which of the following expresses the coordinates of the foci of the conic section shown below? (x-2)^2/4+(y+5)^2/9

Answers

Step-by-step explanation:

[tex] \frac{(x - 2) {}^{2} }{4} + \frac{(y + 5) {}^{2} }{9} = 1[/tex]

This is the equation of the ellipse. Since the denominator is greater for the y values, we have a vertical ellipse. Remember a>b, so a

The formula for the foci of the vertical ellipse is

(h,k+c) and (h,k-c).

where c is

Our center (h,k) is (2, -5)

[tex] {c}^{2} = {a}^{2} - {b}^{2} [/tex]

Here a^2 is 9, b^2 is 4.

[tex] {c}^{2} = 9 - 4[/tex]

[tex] {c}^{2} = 5[/tex]

[tex]c = \sqrt{5} [/tex]

So our foci is

[tex](2, - 5 + \sqrt{5} )[/tex]

and

[tex](2, - 5 - \sqrt{5} )[/tex]

what is the answer please??

Answers

Answer:

See below

Step-by-step explanation:

Area of a circle = pi * r^2

  for this circle  :    Area = 3.142 * 10 * 10 =  314.2 cm^2

The formula for area of a circle is [tex]\pi r^{2}[/tex]

So using pi as 3.142, we can do 3.142 • 100 which is 314.2 when moving place values.

Even though you did not ask, the circumference formula is 2[tex]\pi[/tex]r.

If we were given the same information, where the radius is 10, and we were solving circumference, then we would do 3.14 • 20 = 62.8.

Brainliest if helpful.

Another day, another math problem 3

Answers

Answer:

[tex]2\sqrt{x+1}-3\\domain:[-1, \infty)[/tex]

Step-by-step explanation:

[tex]g(h(x)) = 2(\sqrt{x+1})-3\\g(h(x)) = 2\sqrt{x+1} - 3\\[/tex]

the function is only defined when (x+1) >= 0 (since square root) so the domain is when x >= -1

Simplify
[tex]\left(\frac{123}{321}\right)\left(\frac{456}{654}\right)\left(\frac{789}{987}\right) \left(\frac{123}{321}\right)^{-1}\left(\frac{456}{654}\right)^{-1}\left(\frac{789}{987}\right)^{-1}[/tex]

Answers

Answer:

1

Step-by-step explanation:

using the rule of exponents

[tex](\frac{a}{b}) ^{-1}[/tex] = [tex]\frac{b}{a}[/tex] , then

[tex]\frac{123}{321}[/tex] × [tex]\frac{456}{654}[/tex] × [tex]\frac{789}{987}[/tex] × [tex]\frac{321}{123}[/tex] × [tex]\frac{654}{456}[/tex] × [tex]\frac{987}{789}[/tex]

[ [tex]\frac{123}{321}[/tex] × [tex]\frac{321}{123}[/tex] = 1 , [tex]\frac{456}{654}[/tex] × [tex]\frac{654}{456}[/tex] = 1 , [tex]\frac{789}{987}[/tex] × [tex]\frac{987}{789}[/tex] = 1 ]

= 1 × 1 × 1

= 1

what are the intercepts of x^2 +8x+16​

Answers

Answers:

x intercept = -4

y intercept = 16

The graph is shown below.

=======================================================

Explanation:

To determine the x intercept(s), we set the expression equal to zero and solve for x. In other words, we plug in y = 0 and solve for x.

y = x^2 +8x+16​

0 = x^2 +8x+16​

x^2 +8x+16​ = 0

(x+4)^2 = 0

x+4 = 0

x = -4

There's only one x intercept. It's located at the vertex of the parabola.

The location of the x intercept is the point (-4,0)

------------

To find the y intercept, we plug in x = 0 and compute like so:

y = x^2 +8x+16​

y = (0)^2 +8(0)+16​

y = 16

The y intercept is 16 which is located at (0,16)

Check out the graph below. It was made using the free app Desmos.

Work out the lengths of sides a and b.
Give your answers to 1 decimal place.
triangle a is a right angled triangle with a height | of 8cm base of 5cm and missing hypotenuse
triangle b is a right angled triangle with a height | of 12cm and a hypotenuse of 17cm with a missing base

Answers

The length of the missing sides of triangles A and B are 9.43 cm and 12.04 cm respectively.

Pythagoras theorem

Triangle A:

Height = 8cmBase = 5cmHypotenuse = x cm

Hypotenuse² = height ² + base²

x² = 8² + 5²

= 64 + 25

x² = 89

x = √89

x = 9.43 cm

Triangle B:

Height = 12cmBase = x cmHypotenuse = 17 cm

Hypotenuse² = height ² + base²

17² = 12² + x²

17² - 12² = x²

289 - 144 = x²

145 = x²

x = √145

x = 12.04 cm

Learn more about right triangle:

https://brainly.com/question/2217700

#SPJ1

Where does billy bob johns

Answers

Answer:

in ur moms house

Step-by-step explanation:

Answer:

he doesn't

Step-by-step explanation:

help pls i jusiot jiojfgnio43nfginfmowenfviowngognior3

Answers

Answer:

i dont know

Step-by-step explanation:

sorry please be a bit more clear on what help you would like

The statement “a > 2 and a < 5” is true when a is equal to
A. 5
B. 3
C. 10
D. 2

Answers

Answer:

B

Step-by-step explanation:

because > means greater than or more than

and < means less than

the answer is B because 3 is more than 2 and the same time less than 5

A carpenter wants to cut an 84 inches long board into two-pieces such that one piece is twice as long as the other piece. Set up an equation and find the length of each piece.

Answers

It’s not an equation but the shorter piece would be 28 inches and the longer piece would be 56 inches

Answer:

small piece = 28 inches

large piece = 56 inches

Step-by-step explanation:

The carpenter has one piece of wood 84 inches long. He wants to cut that piece of wood into two pieces. One piece will be 2 times as long as the other. Let's let x = the small piece of wood

2x + x = 84

In this equation, the longer piece is 2 times the shorter piece and the shorter piece is added, which equals 84 inches. Lets solve:

2x + x = 84

3x = 84

3x/3 = 84/3

x = 28

This means that the shorter piece is 28 inches. To find the longer piece, multiply 28 by 2 because the longer piece is twice the shorter piece. Therefore the longer piece is

28 × 2 = 56

Finally, we figured out that the short piece is 28 inches and the longer piece is 56 inches

We can double check this by doing 56 + 28 and making sure it equals 84

If y=4 when x=12, find y when x=-24

Answers

The value of y is -8 if the x  -24 and y varies directly with x, and If y = 4 when x = 12.

What is a proportional relationship?

It is defined as the relationship between two variables when the first variable increases, the second variable also increases according to the constant factor.

The question is incomplete.

The complete question is:

If y varies directly with x, and If y = 4 when x = 12, how do you find y when x = -24?

y ∝ x  (given)

y = kx

k is the constant of proportionality.

4 = 12k  (y = 4, and x = 12)

k = 1/3

y = x/3

Plug x = -24

y = -24/3

y = -8

Thus, the value of y is -8 if the x  -24 and y varies directly with x, and If y = 4 when x = 12.

Learn more about the proportional here:

brainly.com/question/14263719

#SPJ1

Which statements are true? Select three options O The line x = O is perpendicular to the line y = -3 O All lines that are parallel to the y-axis are vertical lines. O All lines that are perpendicular to the x-axis have a slope of O. O The equation of the line parallel to the x-axis that passes through the point (2, -6) is x = 2. O The equation of the line perpendicular to the y-axis that passes through the point (-5, 1) is y = 1

Answers

The statements which are true among the answer choices are;

The line x = O is perpendicular to the line y = -3.All lines that are parallel to the y-axis are vertical lines.

Which statements among the answer choices are true?

In the concept of straight line graphs (linear graphs) the statements are evaluated as follows;

The line x = O is perpendicular to the line y = -3. - This is true because the former is a vertical line while the latter is horizontal.

All lines that are parallel to the y-axis are vertical lines. - True, because the y-axis itself is a vertical line.

All lines that are perpendicular to the x-axis have a slope of O. - False, because all lines perpendicular to the X axis are vertical lines and have do not have slope 0.

The equation of the line parallel to the x-axis that passes through the point (2, -6) is x = 2. - False, because a line parallel to the x-axis is defined by y.

The equation of the line perpendicular to the y-axis that passes through the point (-5, 1) is y = 1. - True.

Read more on parallel and perpendicular lines;

https://brainly.com/question/7197064

#SPJ1

Which expression is equivalent to One-fifth (150 x minus 80 y + 50 minus 50 x minus 25 y + 20)?

20 x minus 21 y + 14

20 x + 11 y + 14

20 x minus 11 y + 6

20 x + 21 y + 6

Answers

Answer:

150 x - 80 y + 50 - 50 x - 25 y + 20

Arranging like terms

150 x - 50 x -25 y - 80y + 50 + 20

100x - 105 y + 70

5 ( 20x - 21y + 14)

Answer:

20x - 21y + 14

Explanation:

[tex]\rightarrow \sf \dfrac{1}{5} (150x - 80y + 50 - 50x - 25y + 20)[/tex]

collect like terms

[tex]\rightarrow \sf \dfrac{1}{5} (150x-50x - 80y -25y + 50 + 20)[/tex]

add/subtract like terms

[tex]\rightarrow \sf \dfrac{1}{5} (100x - 105y+70)[/tex]

distribute inside parenthesis

[tex]\rightarrow \sf \dfrac{1}{5} (100x) + \dfrac{1}{5}( - 105y) + \dfrac{1}{5}(70)[/tex]

simplify the following

[tex]\rightarrow \sf 20x -21y + 14[/tex]

indicate whether the statement is true of false.
please provide a explanation


A linear system with three variables and three equations has a unique solution.

Answers

The statement is false, as the system can have no solutions or infinite solutions.

Is the statement true or false?

The statement says that a system of linear equations with 3 variables and 3 equations has one solution.

If the variables are x, y, and z, then the system can be written as:

[tex]a_1*x + b_1*y + c_1*z = d_1\\\\a_2*x + b_2*y + c_2*z = d_2\\\\a_3*x + b_3*y + c_3*z = d_3[/tex]

Now, the statement is clearly false. Suppose that we have:

[tex]a_1 = a_2 = a_3\\b_1 = b_2 = b_3\\c_1 = c_2 = c_3\\\\d_1 \neq d_2 \neq d_3[/tex]

Then we have 3 parallel equations. Parallel equations never do intercept, then this system has no solutions.

Then there are systems of 3 variables with 3 equations where there are no solutions, so the statement is false.

If you want to learn more about systems of equations:

https://brainly.com/question/13729904

#SPJ1

f(x) = -3 +3
Which graph represents the inverse of function ?
-3
N
-1
4
3-
2-
1-
-2+
-3-
-4+
-5+
W.
Y
54
-1-
4-
3-
2
-4
-2
N
-3
-5+
X.
Y
4-
3-
2-
-1-
3
X
e

Answers

Answer: Y

Step-by-step explanation:

Since f(x) passes through (0,3), its inverse should pass through (3,0).

This eliminates X and Z.

Also, since f(x) has a negative slope, so its inverse should also have a negative slope.

This eliminates W.

The graph of the inverse of function is shown in figure.

We have to given that,

Function is defined as,

f (x) = - 3x + 3

Hence, The inverse of function f (x) is,

f (x) = - 3x + 3

y = - 3x + 3

Solve for x,

3x = y + 3

Divide by 3;

x = (y + 3) / 3

Hence,

f⁻¹ (x) = (x + 3) / 3

f⁻¹ (x) = x/3 + 1

Therefore, The graph of the inverse of function is shown in figure.

Learn more about the function visit:

https://brainly.com/question/11624077

#SPJ7

The following are scores for students in Ms. Kennedy's math class.What is the range of the scores?
451
560
600
422
603
514
A 109
B 149
C 178
D181

Answers

Answer:

The answer will be D. 181

Step-by-step explanation:

603 - 422 = 181

Please mark me as the brainliest.


Finding the Domain and Range of a Graph
Determine the domain and range for the graph below. Write your answer in interval notation.

Answers

Answer:

Domain: [tex][-3, 1)[/tex]Range: [tex][-5, 4][/tex]

Step-by-step explanation:

The domain is the set of x-values and the range is the set of y-values.

Other Questions
Which perspective is most concerned with how individuals interpret their experiences?. Solve the following system to find the point(s) of intersection. y = x - 2x - 3y = 2x - 3 The study of the effects of drugs on the mind and brain is:A. transcranial magnetic stimulation therapy.B. electroconvulsive therapy.C. psychopharmacology.D. psychotherapy.Ask my instructor Solve -6 < 2x + 4 16 HELLPPLet n be the middle number of three consecutive even integers. Write an expression for the sum of these integers.sum of the integers? Precepts about dealing with financial crises that most economists would sign onto are: Justify the following statement - 'You can't always tell if a person has an intellectual disability by looking at them.' Please help me solve this questionn as fast as possible oofff Which of the following statements is correct? An externality is a situation where a project would have an adverse effect on some other part of the firms overall operations. If the project would have a favorable effect on other operations, then this is not an externality. An example of an externality is a situation where a bank opens a new office, and that new office causes deposits in the banks other offices to decline The NPV method automatically deals correctly with externalities, even if the externalities are not specifically identified, but the IRR method does not. This is another reason to favor the NPVBoth the NPV and IRR methods deal correctly with externalities, even if the externalities are not specifically identified. However, the payback method does notIdentifying an externality can never lead to an increase in the calculated NPV Fill in the blank. Micheal Jordan es mas alto (blank) tu. Answer options: A. queB. deC. both que and deD. como Which of the following statements about psychological constructs is true?A.Psychological constructs are easy to define and measure.B.Psychological constructs represent concepts that cannot be observed or measured directly.C.Psychological constructs cannot be researched in psychology because they are unobservable.D.Psychological constructs represent measurable overt behavior.Please select the best answer from the choices provided The table below shows the results of a survey that asked 1052 adults from a certain country if they favored or opposed a tax to fund education. A person is selected atrandom. Complete parts (a) through (c).(a) Find the probability that the person opposed the tax or is female.P(opposed the tax or is female) =(Round to the nearest thousandth as needed.)(b) Find the probability that the person supports the tax or is male.P(supports the tax or is male) = (Round to the nearest thousandth as needed.)(c) Find the probability that the person is not unsure or is female.P(is not unsure or is female)= (Round to the nearest thousandth as needed.) Neither n2 nor o2 are greenhouse gases because GOD PLEASE HELP!Which of these is NOT a key factor that contributed to the financial crisis that started in 2007?bond rating agencies giving CDOs very low ratings, discouraging investors to investhighly leveraged investment accounts that magnified losses when bond prices decreasedexcessive use of subprime mortgages to borrowers with poor creditbanks being incentivized to sell risky mortgages as mortgage-backed securities Is the point (11, 15) on the circle defined by (x-9) + (y-15) = 4?O YesO No What was the purpose of the Munich conference? Which is true regarding the graphed function f(x)? f(O) = 3 f(5) = - 1 f(3) = 2O f(2) = -2 If a utility burned 7.57 million tons of coal that was 2.00% sulfur by weight, how many tons of sulfur dioxide were emitted? Answer in scientific notation. find x if 2+x = -2/3 Marion is going to the hospital for a triple bypass operation. she will have general anesthesia, an intravenous catheter, surgical wounds, and a urinary catheter. which healthcare-associated infection is she at greatest risk for contracting